Divergenz der Streuamplitude auf Baumebene in der Quantenfeldtheorie

Ich studiere eine Spielzeugtheorie in der Quantenfeldtheorie. Es gibt zwei freie Felder: ein echtes massives Skalarfeld ϕ mit Masse M und ein komplexes massives Skalarfeld Ψ mit Masse M .

Sie sind durch gekoppelt

L G Ψ Ψ ϕ
Ich bin mir bewusst, dass dieser Wechselwirkungsterm zu einem Lagrange-Operator führt, der unten unbeschränkt ist, aber es ist nur ein Spielzeugmodell, mit dem ich versuche, die Grundlagen der Quantenfeldtheorie zu verstehen.

Wenn ich nun die Streuamplitude auf Baumebene berechne für Ψ Ψ Ψ Ψ Streuung Ich lande bei a T -Kanaldiagramm und a U -Kanaldiagramm. (In beiden Diagrammen die beiden Ψ Partikel kommen herein, tauschen eine Off-Shell aus ϕ Teilchen und Streuung in ihre Endzustände).

Die Summe dieser beiden Diagramme ergibt das gesamte Matrixelement, das wie folgt aussieht

1 ( T M 2 ) + 1 ( u M 2 )
Ignorieren des Faktors von ich und der Faktor von G 2 . Bei der Berechnung dieser Amplitude habe ich im Rahmen des Massenzentrums/Impulses gearbeitet, wobei die anfänglichen vier Impulse der beiden Teilchen waren ( E ( P ) , 0 , 0 , ± P ) , wobei die letzten vier Impulse der beiden Teilchen sind ( E ( P ) , 0 , ± cos ( θ ) P , ± Sünde ( θ ) P ) . Dadurch wird sichergestellt, dass der Viererimpuls erhalten bleibt.

Mit dieser Parametrisierung T wird 4 ( P Sünde ( θ / 2 ) ) 2 , Und u wird 4 ( P cos ( θ / 2 ) ) 2 . Interessant wird es im Grenzbereich M 0 , dh die Masse des Phi-Teilchens geht gegen Null. In diesem Fall wird die Streuamplitude zu unbegrenzt θ nähert sich Null bzw π . Dies ist meiner Meinung nach ein höchst unphysikalisches Ergebnis.

Ist diese Abweichung das Ergebnis eines grundlegenden Problems mit diesem Spielzeugmodell? Wie können wir dieses Ergebnis verstehen? Was ist die physikalische Interpretation einer divergierenden Streuamplitude? Schlimmer noch ist die Tatsache, dass auch der integrierte/gesamte Wirkungsquerschnitt in all diesen Fällen zu divergieren scheint.

Wird nicht zB ϕ ϕ ϕ Breite geben Γ im Propagator für ϕ , Entfernen der Singularität? Das ist jedoch ein ungewöhnlicher, masseloser Zerfall, also bin ich mir nicht sicher.
Du wirst auch haben Γ / M , was darauf hindeutet, dass die „Resonanz“-Interpretation des Teilchens ϕ ist eine dubiose.
Das hat nichts mit QFT zu tun. Dasselbe passiert, wenn wir die Coulomb-Streuung in E&M oder die klassische Streuung in einem Gravitationspotential untersuchen. Die Kraft hat eine unendliche Reichweite, daher divergiert der Querschnitt in kleinen Winkeln
Als Antwort auf innisfree bin ich mir nicht sicher, ob ein solcher Verfall möglich ist. In dieser Theorie gibt es nur wenige Drei-Punkte-Eckpunkte, und keiner von ihnen, den ich gefunden habe, beinhaltet, dass ein Phi-Partikel hereinkommt und zwei Phi-Partikel herauskommen. Denken Sie daran, dass ich völlig neu bei QFT bin, also ist es möglich, dass ich mich völlig irre.
@chuxley wird es nicht in einer Schleife passieren? Zeichne zB einen Kreis mit a Ψ und setze 3 ϕ externe Leitungen darauf.
Ich kann keinen Weg finden, dies tatsächlich mit einer Schleife zu tun. Das Psi-Feld ist ein komplexes Skalarfeld, daher glaube ich nicht, dass Sie mit einem einzelnen Psi-Partikel tatsächlich einen „Kreis“ zeichnen können. Ich denke, Sie müssten ihn mit einem Psi-Partikel- und einem Anti-Psi-Partikelpaar zeichnen. Ich glaube, ich habe einen Weg gefunden, von einem Phi-Partikel zu drei Partikeln zu gelangen, aber ich werde sehen, ob ich einen Weg finden kann, das Diagramm, das ich später gezeichnet habe, hochzuladen.

Antworten (2)

Was Sie gefunden haben, ist ein sehr einfaches Beispiel einer Infrarot-Divergenz , die alle physikalischen Theorien mit masselosen Teilchen plagt.

Diese Art von Abweichungen sind bereits im klassischen Fall vorhanden (siehe zum Beispiel Ref. 1) und signalisieren normalerweise, dass Sie eine unphysikalische Frage stellen, nicht dass die Theorie selbst unphysikalisch ist. Wenn Sie beispielsweise masselose Teilchen haben, wird es sinnlos, nach der Gesamtzahl von ihnen in einer bestimmten physikalischen Konfiguration zu fragen, während die Frage nach der Gesamtenergie eine gut gestellte Frage ist. Dies spiegelt sich in der Mathematik der Theorie durch Divergenzen wider: Die erste Frage führt zu einem abweichenden Ausdruck, die zweite nicht.

Diese Divergenzen werden dem quantenmechanischen Fall vererbt. Beispielsweise sind IR-Divergenzen in der QED allgegenwärtig, aber es lässt sich nachweisen, dass sich diese Divergenzen für „sinnvolle Wirkungsquerschnitte“, dh für im Sinne des vorigen Absatzes gut gestellte Vorhersagen, immer aufheben. Dies wird manchmal als Bloch-Nordsieck-Auslöschung oder im allgemeineren Fall des Standardmodells als Kinoshita-Lee-Nauenberg-Theorem bezeichnet. Siehe zum Beispiel Refs.2,3,4.

In Ihrem speziellen Fall ist die Analyse etwas einfacher, da Sie es mit einer Skalartheorie im Gegensatz zu einem Eichboson zu tun haben (obwohl man im Allgemeinen Renormierungsschemata auf der Schale aufgeben muss, wenn wir masselose Teilchen haben möchten). Dies wird in Ref. 5 diskutiert.

Für eine weitere Diskussion von Infrarot-Divergenzen siehe Lit. 6-8.

Verweise

  1. Itzykson C., Zuber J.-B. Quantenfeldtheorie, Abschnitt 4-1-2.

  2. Peskin, Schröder. Eine Einführung in die Quantenfeldtheorie, Abschnitt 6.5.

  3. Itzykson C., Zuber J.-B. Quantenfeldtheorie, Abschnitt 8-3-1.

  4. Schwartz MD Quantenfeldtheorie und das Standardmodell, Kapitel 20.

  5. Srednicki M. Quantenfeldtheorie, Kapitel 26 und 27.

  6. Ticciati R. Quantenfeldtheorie für Mathematiker, Abschnitt 19.9.

  7. Pokorski S. Gauge Field Theories, Abschnitte 5.5 und 8.7.

  8. Weinberg S. Quantentheorie der Felder, Vol.1. Stiftungen, Kapitel 13.

Plagen die IR-Divergenzen nicht die physikalischen Theorien? M = Γ = 0 , zB Photon?
@innisfree hmm Ich bin mir nicht sicher, was du meinst. Ja, jede Theorie mit masselosen Teilchen hat potenzielle IR-Abweichungen. QED mit einem masselosen Photon ist da keine Ausnahme.
Danke für deine Antwort. Ich werde mir morgen ein altes Exemplar von Peskin' und Shroesders „Eine Einführung in die Quantenfeldtheorie“ von einem Freund holen, also werde ich mich auf jeden Fall damit befassen. Also nur zur Bestätigung; diese Art von Divergenzen (Infrarot-Divergenzen) allgegenwärtig in Quantenfeldtheorien sind, die masselose Teilchen beinhalten? Auch; vielen Dank, dass Sie die Formatierung meiner Frage so geändert haben, dass die mathematischen Formeln lesbar sind.
@chuxley kein Problem, ich bin froh, dass ich helfen konnte. Und ja, Infrarot-Divergenzen beziehen sich auf masselose Teilchen, und sie sind in der QFT tatsächlich allgegenwärtig. Übrigens: Beachten Sie, dass Srednickis Buch kostenlos online auf seiner Webseite ist, also können Sie dieses Buch auch überprüfen, wenn Sie möchten.
Ich habe gerade nach dem Buch gesucht, danke, dass du mich wissen lässt, dass es online ist. Ich weiß nicht, ob ich im selben Thread eine weitere Frage stellen darf, aber sie hängt mit meiner ursprünglichen Frage zusammen. Kann man in dieser Spielzeugtheorie ein Phi-Teilchen in einer Schleife in zwei Phi-Teilchen zerfallen lassen? Ich versuche herauszufinden, wie es passieren konnte (es kam oben in einem Kommentar auf). Danke :).

Dies ist eine sehr grobe Antwort. Aber das hilft bei deinen Fragen. Sie sehen das Problem mit der obigen Methode, die Sie angedeutet haben, auf Baumebene ψ ψ > ψ ψ der interne Propagator ist ϕ Partikel. Nun sind die Propagatoren für massive und masselose Teilchen sehr unterschiedlich. Für masselose Teilchen ist die Bewegungsgleichung im Impulsraum nicht trivial umkehrbar. Das kann man in obiger Gleichung nicht M = 0 setzen. Für masselos ϕ Sie müssen zuerst den Propagator finden.

Sind Sie sicher, dass sich Propagatoren für massereiche und masselose Skalare nicht trivial unterscheiden? Kannst du sie explizit schreiben?